K
Khách

Hãy nhập câu hỏi của bạn vào đây, nếu là tài khoản VIP, bạn sẽ được ưu tiên trả lời.

27 tháng 6 2017

\(\frac{a}{b+c}+\frac{b}{c+a}+\frac{c}{a+b}\)    \(\ge\frac{3}{2}\)

\(\Leftrightarrow\frac{a}{b+c}+1+\frac{b}{a+c}+1+\frac{c}{a+b}+1\ge\frac{9}{2}\) 

\(\Leftrightarrow\frac{a+b+c}{b+c}+\frac{a+b+c}{a+c}+\frac{a+b+c}{a+b}\ge\frac{9}{2}\) 

\(\Leftrightarrow\left(a+b+c\right)\left(\frac{1}{b+c}+\frac{1}{a+c}+\frac{1}{a+b}\right)\ge\frac{9}{2}\)

\(\Leftrightarrow2\left(a+b+c\right)\left(\frac{1}{b+c}+\frac{1}{a+c}+\frac{1}{a+b}\right)\ge9\) 

thật vậy\(2\left(a+b+c\right)\left(\frac{1}{a+b}+\frac{1}{b+c}+\frac{1}{a+c}\right)\) =\(\left[\left(b+c\right)+\left(a+c\right)+\left(a+b\right)\right]\left(\frac{1}{b+c}+\frac{1}{a+c}+\frac{1}{a+b}\right)\ge9\) (ÁP DỤNG BẤT ĐẲNG THỨC COSI) 

ĐẲNG THỨC CUỐI ĐÚNG SUY RA ĐẲNG THỨC ĐẦU ĐƯỢC CHỨNG MINH

27 tháng 6 2017

ta có \(\frac{a^3}{b}+ab\ge2a^2\)

do đó VT +(ab + bc + ca) \(\ge2a^2+2b^2+2c^2\)

hay VT \(\ge2a^2+2b^2+2c^2-\left(ab+bc+ca\right)\ge a^2+b^2+c^2\) (đpcm).

2 tháng 8 2016

SD bất đẳng thức Côsi:

\(\frac{a^3}{\left(b+2c\right)^2}+\frac{b+2c}{27}+\frac{b+2c}{27}\ge3\sqrt[3]{\frac{a^3}{\left(b+2c\right)^2}.\frac{b+2c}{27}.\frac{b+2c}{27}}=\frac{a}{3}\)

Tương tự rồi cộng lại ta có đpcm

AH
Akai Haruma
Giáo viên
16 tháng 1 2020

Trước tiên, bạn cần lưu ý lần sau đăng bài thì gõ đúng công thức toán!! Người đọc, người giải có thể nhìn cách đăng bài mà bỏ qua bài của bạn.

Lời giải:

Đặt biểu thức vế trái là $P$

Áp dụng BĐT AM-GM:

$a^2+1\geq 2a\Rightarrow a^2+2b+3\geq 2a+2b+2$

$\Rightarrow \frac{a}{a^2+2b+3}\leq \frac{a}{2(a+b+1)}$

Hoàn toàn tương tự với các phân thức còn lại, suy ra:

$P\leq \frac{1}{2}\left(\frac{a}{a+b+1}+\frac{b}{b+c+1}+\frac{c}{c+a+1}\right)(*)$

Áp dụng BĐT Cauchy-Schwarz:

\(\frac{a}{a+b+1}+\frac{b}{b+c+1}+\frac{c}{c+a+1}=\sum \frac{a}{a+b+1}=\sum (1-\frac{b+1}{a+b+1})=3-\sum \frac{(b+1)^2}{(b+1)(a+b+1)}\)

\(\leq 3-\frac{(b+1+c+1+a+1)^2}{a^2+b^2+c^2+ab+bc+ac+3(a+b+c)+3}=3-\frac{(a+b+c+3)^2}{a^2+b^2+c^2+ab+bc+ac+3(a+b+c)+3}(1)\)

Mà:

\((a+b+c+3)^2=a^2+b^2+c^2+2(ab+bc+ac)+9+6(a+b+c)\)

\(=2(a^2+b^2+c^2)+2(ab+bc+ac)+6+6(a+b+c)\) (do $a^2+b^2+c^2=3$)

$=2[a^2+b^2+c^2+ab+bc+ac+3(a+b+c)+3](2)$

Từ $(1);(2)\Rightarrow \sum \frac{a}{a+b+1}\leq 3-2=1(**)$

Từ $(*); (**)\Rightarrow P\leq \frac{1}{2}.1=\frac{1}{2}$ (đpcm)

Dấu "=" xảy ra khi $a=b=c=1$

30 tháng 9 2016

Bài 1. Từ giả thiết suy ra 1-a = b+c và áp dụng \(\left(x+y\right)^2\ge4xy\) 

Ta có : \(4\left(1-a\right)\left(1-b\right)\left(1-c\right)=4\left(b+c\right)\left(1-c\right)\left(1-b\right)\le\left[\left(b+c\right)+\left(1-c\right)\right]^2\left(1-b\right)\)

\(=\left(b+1\right)^2\left(1-b\right)=\left(b+1\right)\left(1-b^2\right)=-b^2\left(b+1\right)+\left(b+1\right)\le b+1=a+2b+c\)

9 tháng 10 2017

Ta có \(a^2b^2+b^2c^2+c^2a^2\geq a^2b^2c^2\Leftrightarrow \frac{1}{a^2}+\frac{1}{b^2}+\frac{1}{c^2}\geq 1\)

BĐT cần chứng minh tương đương với \(\frac{\frac{1}{c^3}}{\frac{1}{a^2}+\frac{1}{b^2}}+\frac{\frac{1}{b^3}}{\frac{1}{a^2}+\frac{1}{c^2}}+\frac{\frac{1}{a^3}}{\frac{1}{b^2}+\frac{1}{c^2}}\geq \frac{\sqrt{3}}{2}\)

Đặt \((\frac{1}{a},\frac{1}{b},\frac{1}{c})=(x,y,z)\). Bài toán trở thành: 

Cho \(x,y,z>0|x^2+y^2+z^2\geq 1\). CMR \(P=\frac{x^3}{y^2+z^2}+\frac{y^3}{z^2+x^2}+\frac{z^3}{x^2+y^2}\geq \frac{\sqrt{3}}{2}\)

Lời giải:

 Áp dụng BĐT Cauchy -Schwarz:

\(P=\frac{x^4}{xy^2+xz^2}+\frac{y^4}{yz^2+yx^2}+\frac{z^4}{zx^2+zy^2}\geq \frac{(x^2+y^2+^2)^2}{x^2(y+z)+y^2(x+z)+z^2(x+y)}\) (1)

Không mất tính tổng quát, giả sử \(x\geq y\geq z\Rightarrow x^2\geq y^2\geq z^2\) 

Và \(y+z\leq z+x\leq x+y\). Khi đó, áp dụng BĐT Chebyshev: 

\(3[x^2(y+z)+y^2(x+z)+z^2(x+y)]\leq (x^2+y^2+z^2)(y+z+x+z+x+y)\)

\(\Leftrightarrow x^2(y+z)+y^2(x+z)+z^2(x+y)\leq \frac{2(x^2+y^2+z^2)(x+y+z)}{3}\)

Theo hệ quả của BĐT Am-Gm thì: \((x+y+z)^2\leq 3(x^2+y^2+z^2)\Rightarrow x+y+z\leq \sqrt{3(x^2+y^2+z^2)}\)

\(\Rightarrow x^2(y+z)+y^2(x+z)+z^2(x+y)\leq \frac{2(x^2+y^2+z^2)\sqrt{3(x^2+y^2+z^2)}}{3}\) (2)

Từ (1),(2) suy ra \(P\geq \frac{3(x^2+y^2+z^2)^2}{2(x^2+y^2+z^2)\sqrt{3(x^2+y^2+z^2)}}=\frac{\sqrt{3(x^2+y^2+z^2)}}{2}\geq \frac{\sqrt{3}}{2}\)

Ta có đpcm

Dáu bằng xảy ra khi \(x=y=z=\frac{1}{\sqrt{3}}\Leftrightarrow a=b=c=\sqrt{3}\)

5 tháng 5 2020

Đặt \(x=\frac{1}{a};y=\frac{1}{b};z=\frac{1}{c}\)

Khi đó giả thiết được viết lại là \(x^2+y^2+z^2\ge1\)và ta cần chứng minh \(\frac{x^3}{y^2+z^2}+\frac{y^3}{z^2+x^2}+\frac{z^3}{x^2+y^2}\ge\frac{\sqrt{3}}{2}\)(*)

Áp dụng BĐT Bunhiacopxki dạng phân thức, ta được:

\(VT_{\left(^∗\right)}=\frac{x^4}{x\left(y^2+z^2\right)}+\frac{y^4}{y\left(z^2+x^2\right)}+\frac{z^4}{z\left(x^2+y^2\right)}\)\(\ge\frac{\left(x^2+y^2+z^2\right)^2}{x\left(y^2+z^2\right)+y\left(z^2+x^2\right)+z\left(x^2+y^2\right)}\)

Đến đây ta đi chứng minh \(\frac{\left(x^2+y^2+z^2\right)^2}{x\left(y^2+z^2\right)+y\left(z^2+x^2\right)+z\left(x^2+y^2\right)}\ge\frac{\sqrt{3}}{2}\)

\(\Leftrightarrow2\left(x^2+y^2+z^2\right)^2\)\(\ge\sqrt{3}\left[x\left(y^2+z^2\right)+y\left(z^2+x^2\right)+z\left(x^2+y^2\right)\right]\)

Ta có: \(x\left(y^2+z^2\right)=\frac{1}{\sqrt{2}}\sqrt{2x^2\left(y^2+z^2\right)\left(y^2+z^2\right)}\)\(\le\frac{1}{\sqrt{2}}\sqrt{\left(\frac{2x^2+y^2+z^2+y^2+z^2}{3}\right)^3}\)

\(=\frac{2\sqrt{3}}{9}\left(x^2+y^2+z^2\right)\sqrt{x^2+y^2+z^2}\)

Tương tự ta có: \(y\left(z^2+x^2\right)\le\frac{2\sqrt{3}}{9}\left(x^2+y^2+z^2\right)\sqrt{x^2+y^2+z^2}\)

\(z\left(x^2+y^2\right)\le\frac{2\sqrt{3}}{9}\left(x^2+y^2+z^2\right)\sqrt{x^2+y^2+z^2}\)

Cộng theo vế của 3 BĐT trên, ta được: 

\(\text{∑}_{cyc}\left[x\left(y^2+z^2\right)\right]\le\frac{2\sqrt{3}}{3}\left(x^2+y^2+z^2\right)\sqrt{x^2+y^2+z^2}\)

\(\Leftrightarrow\sqrt{3}\text{∑}_{cyc}\left[x\left(y^2+z^2\right)\right]\le2\left(x^2+y^2+z^2\right)\sqrt{x^2+y^2+z^2}\)

Cuối cùng ta cần chứng minh được

\(2\left(x^2+y^2+z^2\right)\sqrt{x^2+y^2+z^2}\le2\left(x^2+y^2+z^2\right)^2\)

\(\Leftrightarrow x^2+y^2+z^2\ge1\)(đúng)

Đẳng thức xảy ra khi \(x=y=z=\frac{1}{\sqrt{3}}\Rightarrow a=b=c=\sqrt{3}\)

NV
28 tháng 6 2021

Đề bài sai với \(a=b=c=2\)

28 tháng 6 2021

Có xóa luôn câu hỏi không ạ?